Dadgarnia

New Member
ارسال ها
1,350
لایک ها
1,127
امتیاز
0
#41
پاسخ : ماراتن نظریه اعداد (سطح مقدماتی)

جواب های معادله زیر را بیابید

x,y طبیعی اند
فرض کنید
و داشته باشیم
با قرار دادن این رابطه در صورت سوال داریم:
پس می توانیم از همان اول فرض کنیم
و داریم:

می دانیم که
پس بنابر لم اقلیدس داریم:
با توجه به اینکه 1999 اول است به راحتی می توان چک کرد که معادله جواب طبیعی ندارد.
سوال بعد:
عددی طبیعی است. ثابت کنید اگر
عددی طبیعی باشد آنگاه مربع کامل است.
 
آخرین ویرایش توسط مدیر

darya.f

New Member
ارسال ها
182
لایک ها
114
امتیاز
0
#42
پاسخ : ماراتن نظریه اعداد (سطح مقدماتی)

http://latex.numberempire.com/render?28{n}^{2}+1={m}^{2}\Rightarrow%20%2028{n}^{2}=(m+1)(m-1)%20(M+1,m-1)=1\Rightarrow%20{n}^{2}=(m+1)(m-1)/28%20
مىدونىم که اگر دو عدد که نسبت بهم اول هستند ضربشون مربع کامل شه ,خودشون هم مربع کاملن پس با توزىع 28به مخرج دو پرانتز ىکسرى حالت به وجود مىاد که با چک کردن حکم نتىجه مىشه مثلا ىکى از حالات له اىن شکله:
m+1=2{x}^{2}%20,m-1=14{y}^{2}\Rightarrow%207{y}^{2}={x}^{2}-1%20%20%20%20%20%20%202+2m=2(2{x}^{2}+1-1)=4{x}^{2}\Rightarrow&sig=c5e92f507f2cf07f824fd7bfdea54d46

---- دو نوشته به هم متصل شده است ----

مکعب کامل شود ,تمام p ها را بىابىد
 
آخرین ویرایش توسط مدیر

AHZolfaghari

Well-Known Member
ارسال ها
935
لایک ها
1,654
امتیاز
93
#43
پاسخ : ماراتن نظریه اعداد (سطح مقدماتی)

http://latex.numberempire.com/render?28{n}^{2}+1={m}^{2}\Rightarrow%20%2028{n}^{2}=(m+1)(m-1)%20(M+1,m-1)=1\Rightarrow%20{n}^{2}=(m+1)(m-1)/28%20
مىدونىم که اگر دو عدد که نسبت بهم اول هستند ضربشون مربع کامل شه ,خودشون هم مربع کاملن پس با توزىع 28به مخرج دو پرانتز ىکسرى حالت به وجود مىاد که با چک کردن حکم نتىجه مىشه مثلا ىکى از حالات له اىن شکله:
m+1=2{x}^{2}%20,m-1=14{y}^{2}\Rightarrow%207{y}^{2}={x}^{2}-1%20%20%20%20%20%20%202+2m=2(2{x}^{2}+1-1)=4{x}^{2}\Rightarrow&sig=c5e92f507f2cf07f824fd7bfdea54d46

---- دو نوشته به هم متصل شده است ----

مکعب کامل شود ,تمام p ها را بىابىد


پس دو حالت داره یا p مضرب
هستش یا مضرب



که تناقض است پس :




پس t یا برابر 3 هستش یا از x+2 بزرگ تر هستش
حالتی که از x+2 بزرگتر باشه خیلی راحت به تناقض برمیخوره
پس t=3





اگه اشتباه نکنم سوال بالکان یا جونیور بالکان بوده

سوال بعدی :
کلیه اعداد اول p,q را بیابید که

هروقت سوال حل شد میگم سوال کجا بوده و چه سالی​
 

darya.f

New Member
ارسال ها
182
لایک ها
114
امتیاز
0
#44
پاسخ : ماراتن نظریه اعداد (سطح مقدماتی)




پس دو حالت داره یا p مضرب
هستش یا مضرب



که تناقض است پس :




پس t یا برابر 3 هستش یا از x+2 بزرگ تر هستش
حالتی که از x+2 بزرگتر باشه خیلی راحت به تناقض برمیخوره
پس t=3





اگه اشتباه نکنم سوال بالکان یا جونیور بالکان بوده

سوال بعدی :
کلیه اعداد اول p,q را بیابید که

هروقت سوال حل شد میگم سوال کجا بوده و چه سالی​
[/QUOTE]
 
آخرین ویرایش توسط مدیر

AHZolfaghari

Well-Known Member
ارسال ها
935
لایک ها
1,654
امتیاز
93
#45
پاسخ : ماراتن نظریه اعداد (سطح مقدماتی)




پس دو حالت داره یا p مضرب
هستش یا مضرب



که تناقض است پس :




پس t یا برابر 3 هستش یا از x+2 بزرگ تر هستش
حالتی که از x+2 بزرگتر باشه خیلی راحت به تناقض برمیخوره
پس t=3





اگه اشتباه نکنم سوال بالکان یا جونیور بالکان بوده

سوال بعدی :
کلیه اعداد اول p,q را بیابید که

هروقت سوال حل شد میگم سوال کجا بوده و چه سالی​
[/QUOTE]
راه حل شما درسته . حالتی که p,q اول نباشند هم میشه بررسی کرد . درواقع تو این مساله زیاد اینکه p,q اولند مهم نیست .
سوال TST سال 2013 بوسنی و هرزگوین بوده.
حالا این سوال :
دنباله
رو اینطوری معین می کنیم بطوریکه k یک عدد طبیعی است
برای m,n نابرابر اثبات کنید


---- دو نوشته به هم متصل شده است ----




پس دو حالت داره یا p مضرب
هستش یا مضرب



که تناقض است پس :




پس t یا برابر 3 هستش یا از x+2 بزرگ تر هستش
حالتی که از x+2 بزرگتر باشه خیلی راحت به تناقض برمیخوره
پس t=3





اگه اشتباه نکنم سوال بالکان یا جونیور بالکان بوده

سوال بعدی :
کلیه اعداد اول p,q را بیابید که

هروقت سوال حل شد میگم سوال کجا بوده و چه سالی​
[/QUOTE]
ببخشید جواب هاتون ناقصه
(61و59) و (31و929) رو ننوشتید البته دیگه حواستون به تقارن باشه دیگه که برعکس شون هم جواب هست یعنی 6 تا جواب داره
 

darya.f

New Member
ارسال ها
182
لایک ها
114
امتیاز
0
#46
پاسخ : ماراتن نظریه اعداد (سطح مقدماتی)

راه حل شما درسته . حالتی که p,q اول نباشند هم میشه بررسی کرد . درواقع تو این مساله زیاد اینکه p,q اولند مهم نیست .
سوال TST سال 2013 بوسنی و هرزگوین بوده.
حالا این سوال :
دنباله
رو اینطوری معین می کنیم بطوریکه k یک عدد طبیعی است
برای m,n نابرابر اثبات کنید


---- دو نوشته به هم متصل شده است ----

[/QUOTE]
ببخشید جواب هاتون ناقصه
(61و59) و (31و929) رو ننوشتید البته دیگه حواستون به تقارن باشه دیگه که برعکس شون هم جواب هست یعنی 6 تا جواب داره[/QUOTE]
بله بله...حواسم نبود به اونا...:3:
 

math1998

New Member
ارسال ها
336
لایک ها
224
امتیاز
0
#47
پاسخ : ماراتن نظریه اعداد (سطح مقدماتی)

راه حل شما درسته . حالتی که p,q اول نباشند هم میشه بررسی کرد . درواقع تو این مساله زیاد اینکه p,q اولند مهم نیست .
سوال TST سال 2013 بوسنی و هرزگوین بوده.
حالا این سوال :
دنباله
رو اینطوری معین می کنیم بطوریکه k یک عدد طبیعی است
برای m,n نابرابر اثبات کنید


---- دو نوشته به هم متصل شده است ----

[/QUOTE]
ببخشید جواب هاتون ناقصه
(61و59) و (31و929) رو ننوشتید البته دیگه حواستون به تقارن باشه دیگه که برعکس شون هم جواب هست یعنی 6 تا جواب داره[/QUOTE]

ابتدا با استقرا ثابت کنید به ازای هر
سپس ثابت کنید به ازای هر
داریم
و سپس ثابت کنید
 
آخرین ویرایش توسط مدیر
ارسال ها
337
لایک ها
82
امتیاز
0
#48
پاسخ : ماراتن نظریه اعداد (سطح مقدماتی)

خوبه که ماراتن گرم شد اگه راضی باشین سوال بعد من میذارم چون یه یه روزی اصن فعالیت نداشتم
تمام اعداد طبیعی را بیابید که
 

AHZolfaghari

Well-Known Member
ارسال ها
935
لایک ها
1,654
امتیاز
93
#49
پاسخ : ماراتن نظریه اعداد (سطح مقدماتی)

راه حل شما درسته . حالتی که p,q اول نباشند هم میشه بررسی کرد . درواقع تو این مساله زیاد اینکه p,q اولند مهم نیست .
سوال TST سال 2013 بوسنی و هرزگوین بوده.
حالا این سوال :
دنباله
رو اینطوری معین می کنیم بطوریکه k یک عدد طبیعی است
برای m,n نابرابر اثبات کنید


---- دو نوشته به هم متصل شده است ----
ببخشید جواب هاتون ناقصه
(61و59) و (31و929) رو ننوشتید البته دیگه حواستون به تقارن باشه دیگه که برعکس شون هم جواب هست یعنی 6 تا جواب داره[/QUOTE]

ابتدا با استقرا ثابت کنید به ازای هر
سپس ثابت کنید به ازای هر
داریم
و سپس ثابت کنید
[/QUOTE]
میشه با یه کم بازی با رابطه اصلی اثبات کرد جمله n ام میشه حاصلجمع k با حاصلضرب جملات قبل از آن تا جمله اول بیان ریاضیش اینه :

پس ب م م هردوتایی k رو میشماره .و خیلی راحت می فهمیم که تمامی جملات دنباله به پیمانه k یک هستند پس ب م م هردوتایی میشه یک !

---- دو نوشته به هم متصل شده است ----

خوبه که ماراتن گرم شد اگه راضی باشین سوال بعد من میذارم چون یه یه روزی اصن فعالیت نداشتم
تمام اعداد طبیعی را بیابید که




اگر d و 1997 نسبت به هم اول باشند پس می توان نتیجه گرفت

هم چنین داشتیک
پس دو حالت دارید یا
یا

حالا تو معادله اصلی جایگذاری می کنیم تو حالت اول این بدست میاد

که
پس d برابر 5 میشه پس x=170 , y=145
حالت دوم به این معادله تبدیل میشه :

که این هم فکر میکنم فقط این جواب رو داره

پس ب م م میشه29 پس

اما حالتی که d , 1997 ب م م شون 1 نشه پس 1997 هستش پس 1997 x رو میشماره اما میشه اثبات کرد که x از 1997 کم تره پس تناقض پس دوجواب مذکور جواب مساله اند
 

math1998

New Member
ارسال ها
336
لایک ها
224
امتیاز
0
#50
پاسخ : ماراتن نظریه اعداد (سطح مقدماتی)

سوال بعد. همه ی اعداد طبیعی
را پیدا کنید که
 

Dadgarnia

New Member
ارسال ها
1,350
لایک ها
1,127
امتیاز
0
#51
پاسخ : ماراتن نظریه اعداد (سطح مقدماتی)

سوال بعد. همه ی اعداد طبیعی
را پیدا کنید که
فرض مي كنيم
و
و داشته باشيم
پس داريم:



حالت ديگر هم به همين ترتيب بدست مي آيد.
سوال بعد:
اگر
اعدادي طبيعي باشند كه
ثابت كنيد k مربع كامل است.
 
آخرین ویرایش توسط مدیر

math1998

New Member
ارسال ها
336
لایک ها
224
امتیاز
0
#52
پاسخ : ماراتن نظریه اعداد (سطح مقدماتی)

فرض مي كنيم
و
و داشته باشيم
پس داريم:



حالت ديگر هم به همين ترتيب بدست مي آيد

بسیار قشنگ
 

AHZolfaghari

Well-Known Member
ارسال ها
935
لایک ها
1,654
امتیاز
93
#53
پاسخ : ماراتن نظریه اعداد (سطح مقدماتی)

فرض مي كنيم
و
و داشته باشيم
پس داريم:



حالت ديگر هم به همين ترتيب بدست مي آيد.
سوال بعد:
اگر
اعدادي طبيعي باشند كه
ثابت كنيد k مربع كامل است.
راه حلش فکر میکنم با نزول نامتناهی باشه
 

Dadgarnia

New Member
ارسال ها
1,350
لایک ها
1,127
امتیاز
0
#54
پاسخ : ماراتن نظریه اعداد (سطح مقدماتی)

راه حلش فکر میکنم با نزول نامتناهی باشه
تقريبا. سؤال ساده ايه فقط كافيه حالت
رو رد كنيد و فرض كنيد a,b اي موجود باشه كه k مربع كامل نباشه و
كوچكترين مقدار ممكن باشه و به تناقض برسيد.
 

AHZolfaghari

Well-Known Member
ارسال ها
935
لایک ها
1,654
امتیاز
93
#55
پاسخ : ماراتن نظریه اعداد (سطح مقدماتی)

تمام n های طبیعی را بیابید بطوریکه
مربع کامل شود .
 

Dadgarnia

New Member
ارسال ها
1,350
لایک ها
1,127
امتیاز
0
#56
پاسخ : ماراتن نظریه اعداد (سطح مقدماتی)

تمام n های طبیعی را بیابید بطوریکه
مربع کامل شود .
فرض مي كنيم
دو طرف رو به پيمانه ي 3 در نظر مي گيريم:

با قرار دادن اين رابطه در رابطه ي بالا داريم:
حالا چهار حالت رو در نظر مي گيريم:
١-

داريم:

به ازاي k=1 رابطه ي بالا برقرار مي باشد كه جواب n=2 بدست مي آيد. براي k بزگتر از يك داريم:



كه به وضوح تناقض است.
٢-

داريم:

كه تناقض است. دو حالت ديگر هم به همين ترتيب بررسي مي شود كه تنها جواب n=2 است.
 

AHZolfaghari

Well-Known Member
ارسال ها
935
لایک ها
1,654
امتیاز
93
#57
پاسخ : ماراتن نظریه اعداد (سطح مقدماتی)

فرض مي كنيم
دو طرف رو به پيمانه ي 3 در نظر مي گيريم:

با قرار دادن اين رابطه در رابطه ي بالا داريم:
حالا چهار حالت رو در نظر مي گيريم:
١-

داريم:

به ازاي k=1 رابطه ي بالا برقرار مي باشد كه جواب n=2 بدست مي آيد. براي k بزگتر از يك داريم:



كه به وضوح تناقض است.
٢-

داريم:

كه تناقض است. دو حالت ديگر هم به همين ترتيب بررسي مي شود كه تنها جواب n=2 است.
بله درست است کاملا . خالی از لطف نیست اگه راه حلی که باهاش حل کردم این سوال رو هم بذارم:





اگه n یه عامل p فرد داشته باشه در اون صورت
که تناقض داره چون عدد مربع به پیمانه 3 دو حالت داره یک یا صفر که در هرصورت
بر سه

بخش پذیر نمیشه . پس n عامل فرد نداره پس n توانی از دو هستش اگه بزرگتر مساوی چهار باشه در اون صورت رقم یکان
برابر سه میشه که امکان نداره چون رقم

یکان مربع کامل همواره 0 یا 1 یا 4 یا 5 یا 6 یا 9 هستش پس n باید دو باشه که اگه چک کنیم نتیجه می شه که برقرار پس تنها جواب n=2 هستش .

---- دو نوشته به هم متصل شده است ----

سوال بعد :

همه x , y های طبیعی را بیابید که
. البته زیاد سخت نیست
 

Dadgarnia

New Member
ارسال ها
1,350
لایک ها
1,127
امتیاز
0
#58
پاسخ : ماراتن نظریه اعداد (سطح مقدماتی)

واضحه كه
از طرفي داريم:

كه تناقض است پس جوابي وجود نداره.
 

AHZolfaghari

Well-Known Member
ارسال ها
935
لایک ها
1,654
امتیاز
93
#59
پاسخ : ماراتن نظریه اعداد (سطح مقدماتی)

واضحه كه
از طرفي داريم:

كه تناقض است پس جوابي وجود نداره.
درسته

اثبات کنید به ازای هر m,n طبیعی که m<n داریم :
طبیعی است.
 

Dadgarnia

New Member
ارسال ها
1,350
لایک ها
1,127
امتیاز
0
#60
پاسخ : ماراتن نظریه اعداد (سطح مقدماتی)

درسته

اثبات کنید به ازای هر m,n طبیعی که m<n داریم :
طبیعی است.
فرض مي كنيم:
ثابت مي كنيم توان هر عدد اولي كه مخرج رو ميشماره از توان همون عدد توي صورت كمتره. داريم:

پس كافي است براي هر
ثابت كنيم:

اگر
در اين صورت حكم ثابت شده است پس فرض كنيد i اي موجود باشه كه اين رابطه بر قرار نباشه در اين صورت مي دانيم كه توان
در كسر
برابر با
و در كسر
برابر با
است كه حاصل ضرب اين دو توان صفر را به ما مي دهد حالا كافي است توان
در عبارت
را مورد بررسي قرار بدهيم كه در اين عبارت هم حكم بر قرار مي بأشد پس در كل حكم ثابت مي شود.
 
بالا